Master Index Notation: How to Ensure Accuracy in Calculations

In summary: Yes, it does not matter the order, the indices just need to be the same length.Thank you for your help.
  • #1
Petraa
21
0
I'm not sure if this step on my calculation is correct or not.

[tex]
A=i\left(M_{\alpha}^{\nu}M_{\beta\nu}-M_{\beta}^{\nu}M_{\alpha\nu}\right)=i\left(M_{\beta}^{\nu}\delta_{\alpha}^{\beta}M_{\beta\nu}-M_{\beta}^{\nu}M_{\alpha\nu}\right)=i\left(M_{\beta}^{\nu}M_{\alpha\nu}-M_{\beta}^{\nu}M_{\alpha\nu}\right)=0[/tex]

I just change the label in the first generator with the delta and then contract the delta with the other generator. Is it 100% rigorous ?

Thank you!
 
Physics news on Phys.org
  • #2
Petraa said:
I'm not sure if this step on my calculation is correct or not.

[tex]
A=i\left(M_{\alpha}^{\nu}M_{\beta\nu}-M_{\beta}^{\nu}M_{\alpha\nu}\right)=i\left(M_{\beta}^{\nu}\delta_{\alpha}^{\beta}M_{\beta\nu}-M_{\beta}^{\nu}M_{\alpha\nu}\right)=i\left(M_{\beta}^{\nu}M_{\alpha\nu}-M_{\beta}^{\nu}M_{\alpha\nu}\right)=0[/tex]

I just change the label in the first generator with the delta and then contract the delta with the other generator. Is it 100% rigorous ?

Thank you!

You will have to modify this, you are not allowed to call a summation index the same thing as a free index and then change which is the free index. Generally, if you have more than two of the samme index, you have done something naughty.
 
  • #3
Thank you for your fast response. The problem is that this thing should be zero ... if it is not zero then there must be an error in the previous calculations. Is A=0 ? And why?
 
  • #4
What is M? Does it have some additional properties such as symmetry (as suggested by not caring too much about index ordering with the raised index)? If it is, what stops you from simply raising the nu on one M and lowering it on the other?
 
  • Like
Likes Mentz114
  • #5
M is total antisymetric. M are the generators of the lorentz group. The index order does matter but I have written it too fast in tex.
 
  • #6
I still suspect it is a matter of simply changing which summation index is up and which is down ...
 
  • #7
The expression is [tex]
A=i\left(M_{\thinspace\thinspace\alpha}^{\nu}M_{\beta\nu}-M_{\thinspace\thinspace\beta}^{\nu}M_{\alpha\nu}\right);\thinspace\thinspace\thinspace\thinspace\thinspace\left(M_{\alpha\beta}\right)_{\thinspace\thinspace\nu}^{\mu}=\delta_{\alpha}^{\mu}g_{\beta\nu}-\delta_{\beta}^{\mu}g_{\alpha\nu}[/tex]

I don't see how this two terms equal to zero
 
  • #8
Are you familiar with how to raise and lower indices?
 
  • #9
Yes with the metric. I've done it to get to this last result.
Do you mean things like ...

[tex]
A^{i}=g^{ij}A_{j};A_{i}=g_{ij}A^{j};g^{ij}g_{ib}=g_{\thinspace\thinspace b}^{j}=\delta_{b}^{j}
[/tex]

and this sort of stuff ?
 
  • #10
Exactly, so how does ##M^\nu_{\phantom\nu \alpha}## relate to ##M_{\nu\alpha}##?
 
  • #11
I think its

[tex]
M_{\nu\alpha}=g_{\nu\gamma}M_{\thinspace\thinspace\alpha}^{\gamma};M_{\thinspace\thinspace\alpha}^{\nu}=g^{\nu\gamma}M_{\gamma\alpha}[/tex]
 
  • #12
So what do you get if you apply the latter of these relations to your expression? (And switch the names of the summation indices in one of the terms...)
 
  • #13
I know what are trying to suggest me but I've tried that and I don't see why it should cancel. I feel so stupid right now :P. For example

[tex]A=i\left(M_{\thinspace\alpha}^{\nu}M_{\beta\nu}-M_{\thinspace\beta}^{\nu}M_{\alpha\nu}\right)=i\left(g^{\nu\xi}M_{\xi\alpha}M_{\beta\nu}-M_{\thinspace\beta}^{\nu}M_{\alpha\nu}\right)=i\left(M_{\xi\alpha}M_{\beta}^{\thinspace\thinspace\xi}-M_{\thinspace\beta}^{\nu}M_{\alpha\nu}\right)[/tex]

Pffff Sorry for wasting ur time
 
  • #14
Do it for both terms! :smile:

Edit: Or not even that. Rename the summation index in the first term from ##\xi## to ##\nu## and use the antisymmetry of M.
 
  • #15
Yes, this is exactly the part that I do not understand. I thought that this procedure was incorrect...

M is antisymmetric so
[tex]
M_{ij}=-M_{ji}
[/tex]
I guess that
[tex]
M_{\thinspace\thinspace j}^{i}=-M_{\thinspace\thinspace i}^{j}
[/tex]
[tex]A=i\left(M_{\xi\alpha}M_{\beta}^{\thinspace\thinspace\xi}-M_{\thinspace\beta}^{\nu}M_{\alpha\nu}\right)=i\left(M_{\nu\alpha}M_{\beta}^{\thinspace\thinspace\nu}-M_{\thinspace\thinspace\beta}^{\nu}M_{\alpha\nu}\right)=i\left(-M_{\alpha\nu}M_{\beta}^{\thinspace\thinspace\nu}-M_{\thinspace\thinspace\beta}^{\nu}M_{\alpha\nu}\right)[/tex]

And this is the farthest I could arrive before posting this thread and I don't get why this is zero. M generators does not even commute so ...

One thing that I'm not 100% sure arei the indexs on the generators. They are not matrix indexs so it does not matter the order, right ?

[tex]M_{\thinspace\thinspace j}^{i}=M_{ j}^{\thinspace\thinspace i}[/tex]
 
  • #16
Petraa said:
The expression is [tex]
A=i\left(M_{\thinspace\thinspace\alpha}^{\nu}M_{\beta\nu}-M_{\thinspace\thinspace\beta}^{\nu}M_{\alpha\nu}\right);\thinspace\thinspace\thinspace\thinspace\thinspace\left(M_{\alpha\beta}\right)_{\thinspace\thinspace\nu}^{\mu}=\delta_{\alpha}^{\mu}g_{\beta\nu}-\delta_{\beta}^{\mu}g_{\alpha\nu}[/tex]

I don't see how this two terms equal to zero

Assuming you meant to write [tex]A_{\alpha \beta} = i \left( M^{\nu}{}_{\alpha} \ M_{\beta \nu} - M^{\nu}{}_{\beta} \ M_{\alpha \nu} \right) ,[/tex] then by lowering the index [itex]\nu[/itex] [tex]A_{\alpha \beta} = i \left( \eta^{\nu \rho} M_{\rho \alpha} \ M_{\beta \nu} - \eta^{\nu \rho} M_{\rho \beta} \ M_{\alpha \nu} \right) .[/tex] In the second term, change the dummy indices [itex]\nu \leftrightarrow \rho[/itex] and find [tex]A_{\alpha \beta} = - i \eta^{\nu \rho} [ M_{\alpha \rho} , M_{\beta \nu} ] .[/tex] Now, using the Lorentz algebra [tex]i [M_{\alpha \rho} , M_{\beta \nu} ] = \eta_{\alpha \beta} M_{\rho \nu} - \eta_{\rho \beta} M_{\alpha \nu} + \eta_{\alpha \nu} M_{\beta \rho} - \eta_{\rho \nu} M_{\beta \alpha} ,[/tex] you find [tex]A_{\alpha \beta} = ( 2 - n ) M_{\alpha \beta} ,[/tex] where [itex]n[/itex] is the dimension of space-time.
Clearly, [itex]A_{\alpha \beta} = 0[/itex], only for [itex]n = 2[/itex], i.e., in 2-dimensional space-time.
However, if by [itex]A[/itex] you meant the trace [itex]A = \eta^{\alpha \beta} A_{\alpha \beta}[/itex], then yes [itex]A = 0[/itex] identically.

Sam
 
  • #17
if ##M## is antisymmetric then ( I believe) ##M_{\thinspace\thinspace\alpha}^{\nu}M_{\beta\nu}## is symmetric in ##\alpha,\ \beta##
 
  • #18
What I am trying to prove is that the commutator of the operator W^2 constructed from the Pauli-Lubanski pseudo vector is a Casimir operator for the poincare group.

The whole commutator [W^2,M_ij] splits up into two parts, my textbook says that the first part should be zero and I've revised multiple times my calculations and I always get what I have written in my first post...

@Mentz114 if you are correct then of course is 0 but I can not prove it.
 
  • #19
Petraa said:
@Mentz114 if you are correct then of course is 0 but I can not prove it.

It is really only a matter of doing what I proposed you to do. I suggest first switching the indices of the M with both indices down, then you switch which index is up and which is down, then you switch the indices of the other. You can never have ##M^\mu_{\phantom\mu \nu} = -M^{\phantom \mu \nu}_\mu##, the different sides have different indices contra/covariant.
 
  • #20
Orodruin said:
It is really only a matter of doing what I proposed you to do. I suggest first switching the indices of the M with both indices down, then you switch which index is up and which is down, then you switch the indices of the other. You can never have ##M^\mu_{\phantom\mu \nu} = -M^{\phantom \mu \nu}_\mu##, the different sides have different indices contra/covariant.

If I play with the indexs, lowering them etc. I just walk in circles. For example I get

[tex]i\left(M_{\thinspace\alpha}^{\nu}M_{\beta\nu}-M_{\thinspace\beta}^{\nu}M_{\alpha\nu}\right)=i\left(g^{\nu\xi}M_{\xi\alpha}M_{\beta\nu}-g^{\nu\xi}M_{\xi\beta}M_{\alpha\nu}\right)=i\left(M_{\xi\alpha}M_{\beta}^{\thinspace\thinspace\xi}-M_{\xi\beta}M_{\alpha}^{\thinspace\thinspace\xi}\right)[/tex]
 
  • #21
Petraa said:
If I play with the indexs, lowering them etc. I just walk in circles. For example I get

[tex]i\left(M_{\thinspace\alpha}^{\nu}M_{\beta\nu}-M_{\thinspace\beta}^{\nu}M_{\alpha\nu}\right)=i\left(g^{\nu\xi}M_{\xi\alpha}M_{\beta\nu}-g^{\nu\xi}M_{\xi\beta}M_{\alpha\nu}\right)=i\left(M_{\xi\alpha}M_{\beta}^{\thinspace\thinspace\xi}-M_{\xi\beta}M_{\alpha}^{\thinspace\thinspace\xi}\right)[/tex]

I also told you to switch the names of the summation indices in one of the terms ... Do that and factorise out ##g^{\nu\xi}## from the expression ...

Edit: And use that the metric is symmetric ...
 
  • #22
Mentz114 said:
if ##M## is antisymmetric then ( I believe) ##M_{\thinspace\thinspace\alpha}^{\nu}M_{\beta\nu}## is symmetric in ##\alpha,\ \beta##

No, you are very wrong. I will try to repeat what I have already done in my previous post. [tex]M^{\nu}{}_{\alpha} M_{\beta \nu} = g^{\nu \mu} M_{\mu \alpha} M_{\beta \nu} = g^{\nu \mu} ( - M_{\alpha \mu} ) ( - M_{\nu \beta} ) = g^{\nu \mu} M_{\alpha \mu} M_{\nu \beta} . \ \ (1)[/tex] [tex]M^{\nu}{}_{\beta} M_{\alpha \nu} = g^{\rho \sigma} M_{\rho \beta} M_{\alpha \sigma} = g^{\nu \mu} M_{\nu \beta} M_{\alpha \mu} . \ \ \ \ \ \ \ \ \ \ \ \ \ \ \ (2)[/tex] Now, subtract (2) from (1) [tex]M^{\nu}{}_{\alpha} M_{\beta \nu} - M^{\nu}{}_{\beta} M_{\alpha \nu} = g^{\nu \mu} \left( M_{\alpha \mu} M_{\nu \beta} - M_{\nu \beta} M_{\alpha \mu} \right) = g^{\nu \mu} [ M_{\alpha \mu} , M_{\nu \beta} ] \neq 0 .[/tex]
 
  • #23
Petraa said:
If I play with the indexs, lowering them etc. I just walk in circles. For example I get

[tex]i\left(M_{\thinspace\alpha}^{\nu}M_{\beta\nu}-M_{\thinspace\beta}^{\nu}M_{\alpha\nu}\right)=i\left(g^{\nu\xi}M_{\xi\alpha}M_{\beta\nu}-g^{\nu\xi}M_{\xi\beta}M_{\alpha\nu}\right)=i\left(M_{\xi\alpha}M_{\beta}^{\thinspace\thinspace\xi}-M_{\xi\beta}M_{\alpha}^{\thinspace\thinspace\xi}\right)[/tex]

Did you read my first post? The expression you wrote IS NOT zero.
 
  • #24
Petraa said:
What I am trying to prove is that the commutator of the operator W^2 constructed from the Pauli-Lubanski pseudo vector is a Casimir operator for the poincare group.


First, you need to show that [itex]W_{\mu}[/itex] is a Lorentz vector, i.e. you need to prove that [tex][ M_{\mu \nu} , W_{\rho} ] = i \left( g_{\nu \rho} W_{\mu} - g_{\mu \rho} W_{\nu} \right) . \ \ \ (1)[/tex] Once you have done that, the rest is easy. Use [tex][ M_{\mu \nu} , W^{2} ] = g^{\rho \sigma} [ M_{\mu \nu} , W_{\rho} ] W_{\sigma} + g^{\rho \sigma} W_{\rho} [ M_{\mu \nu} , W_{\sigma} ] . \ \ (2)[/tex] Now, insert (1) in (2) and do the trivial algebra, you find [tex][ M_{\mu \nu} , W^{2} ] = 0 .[/tex] Proving [itex][ W^{2} , P_{\mu} ] = 0[/itex] is even easier, because [tex][ W_{\rho} , P_{\mu} ] = \frac{1}{2} \epsilon_{\rho \sigma \alpha \beta} g_{\mu \nu} P^{\sigma} [ M^{\alpha \beta} , P^{\nu} ] = \frac{i}{2} \epsilon_{\rho \sigma \alpha \mu} [ P^{\sigma} , P^{\alpha} ] = 0 .[/tex]
 
  • #25
samalkhaiat said:
No, you are very wrong. I will try to repeat what I have already done in my previous post. [tex]M^{\nu}{}_{\alpha} M_{\beta \nu} = g^{\nu \mu} M_{\mu \alpha} M_{\beta \nu} = g^{\nu \mu} ( - M_{\alpha \mu} ) ( - M_{\nu \beta} ) = g^{\nu \mu} M_{\alpha \mu} M_{\nu \beta} . \ \ (1)[/tex] [tex]M^{\nu}{}_{\beta} M_{\alpha \nu} = g^{\rho \sigma} M_{\rho \beta} M_{\alpha \sigma} = g^{\nu \mu} M_{\nu \beta} M_{\alpha \mu} . \ \ \ \ \ \ \ \ \ \ \ \ \ \ \ (2)[/tex] Now, subtract (2) from (1) [tex]M^{\nu}{}_{\alpha} M_{\beta \nu} - M^{\nu}{}_{\beta} M_{\alpha \nu} = g^{\nu \mu} \left( M_{\alpha \mu} M_{\nu \beta} - M_{\nu \beta} M_{\alpha \mu} \right) = g^{\nu \mu} [ M_{\alpha \mu} , M_{\nu \beta} ] \neq 0 .[/tex]

What about the definition of the EM energy-momentun tensor ##T^{\mu\nu}={F^\mu}_\rho F^{\rho\nu}## which alway gives a symmetric T ?
I checked this and it is definitely symmetric in ##\nu\mu## but not if ##\nu## is interchanged with another index (obviously) .
 
  • #26
Mentz114 said:
What about the definition of the EM energy-momentun tensor ##T^{\mu\nu}={F^\mu}_\rho F^{\rho\nu}## which alway gives a symmetric T ?
I checked this and it is definitely symmetric in ##\nu\mu## but not if ##\nu## is interchanged with another index (obviously) .

This is true when ##F^{\mu\nu}## is a number, meaning that the ##F##s commute. It will no longer be true if it is a group element as given by the last equation of post #22 (note that if the ##M## were numbers, then the commutator would be zero).
 
  • #27
Petraa said:
What I am trying to prove is that the commutator of the operator W^2 constructed from the Pauli-Lubanski pseudo vector is a Casimir operator for the poincare group.

The whole commutator [W^2,M_ij] splits up into two parts, my textbook says that the first part should be zero and I've revised multiple times my calculations and I always get what I have written in my first post...

@Mentz114 if you are correct then of course is 0 but I can not prove it.

Right now I cannot prove it is always true.
 
  • #28
Orodruin said:
This is true when ##F^{\mu\nu}## is a number, meaning that the ##F##s commute. It will no longer be true if it is a group element as given by the last equation of post #22 (note that if the ##M## were numbers, then the commutator would be zero).

OK, thanks. I forget that those commutators have implied indexes.
 
  • #29
samalkhaiat said:
First, you need to show that [itex]W_{\mu}[/itex] is a Lorentz vector, i.e. you need to prove that [tex][ M_{\mu \nu} , W_{\rho} ] = i \left( g_{\nu \rho} W_{\mu} - g_{\mu \rho} W_{\nu} \right) . \ \ \ (1)[/tex]
The proof rests on the following facts:
(1) [itex]P^{\mu}[/itex] is a vector operator, i.e. it transforms according to [tex]U^{\dagger} ( \Lambda ) P^{\mu} U ( \Lambda ) = \Lambda^{\mu}{}_{\nu} \ P^{\nu} , \ \ \ \ (1)[/tex] where [tex]U ( \Lambda ) = e^{- \frac{i}{2} \omega_{\rho \sigma} M^{\rho \sigma}} ,[/tex] is (infinite-dimensional) unitary representation of the Lorentz group on Hilbert space.
(2) The Lorentz’s generators [itex]M^{\mu \nu}[/itex] transform like tensor operators [tex]U^{\dagger} ( \Lambda ) M^{\mu \nu} U ( \Lambda ) = \Lambda^{\mu}{}_{\rho} \ \Lambda^{\nu}{}_{\sigma} \ M^{\rho \sigma} . \ \ \ \ (2)[/tex]
(3) [itex]\epsilon[/itex] is an invariant tensor: [tex]\epsilon_{\mu \nu \rho \sigma} \Lambda^{\mu}{}_{\alpha} \Lambda^{\nu}{}_{\beta} \Lambda^{\rho}{}_{\gamma} \Lambda^{\sigma}{}_{\tau} = \epsilon_{\alpha \beta \gamma \tau} \ \det | \Lambda | = \epsilon_{\alpha \beta \gamma \tau} ,[/tex] or [tex]\epsilon_{\lambda \nu \rho \sigma} \Lambda^{\nu}{}_{\beta} \Lambda^{\rho}{}_{\gamma} \Lambda^{\sigma}{}_{\tau} = \Lambda^{\alpha}{}_{\lambda} \epsilon_{\alpha \beta \gamma \tau} . \ \ \ (3)[/tex]
So, let us write [tex]U^{\dagger} ( \Lambda ) W_{\mu} U ( \Lambda ) = \frac{1}{2} \epsilon_{\mu \nu \rho \sigma} U^{\dagger} P^{\nu} U U^{\dagger} M^{\rho \sigma} U .[/tex] Using (1), (2) and (3) we find [tex]U^{\dagger} W_{\mu} U = \Lambda^{\nu}{}_{\mu} W_{\nu} .[/tex] This is the finite version of the wanted commutation relation. You can obtain it in the first order approximation [tex]U = 1 - \frac{i}{2} \omega_{\mu \nu} M^{\mu \nu} , \ \ \ \ \Lambda^{\mu}{}_{\nu} = \delta^{\mu}{}_{\nu} + \omega^{\mu}{}_{\nu} .[/tex] One last important commutator that you can prove is [tex][W_{\mu} , W_{\nu}] = i \epsilon_{\mu \nu \rho}{}^{\sigma} P^{\rho} W_{\sigma} .[/tex] From this commutator, you can show that the vector [itex]W_{\mu}[/itex] determines an intrinsic angular momentum, i.e. spin of the representation.
Sam
 

1. What is master index notation?

Master index notation is a mathematical notation used to simplify and ensure accuracy in calculations involving multiple indices or variables.

2. How is master index notation different from standard index notation?

Master index notation uses a single master index to represent multiple indices or variables, while standard index notation uses separate indices for each variable. This makes calculations more efficient and reduces the chances of errors.

3. How do I use master index notation in calculations?

To use master index notation, first identify all the indices or variables involved in the calculation. Then, assign each index a number or letter and create a master index using these numbers or letters. Finally, use the master index in the calculation instead of the individual indices.

4. What are the benefits of using master index notation in calculations?

Master index notation reduces the chances of error in calculations, as it simplifies and streamlines the process by using a single master index. It also allows for easier manipulation of equations and simplification of complex expressions.

5. Are there any limitations to using master index notation?

While master index notation can be useful in many cases, it may not be appropriate for all types of calculations. It is most commonly used in physics and engineering, but may not be as useful for other fields of study. In addition, it may take some time and practice to become comfortable with using master index notation in calculations.

Similar threads

  • Advanced Physics Homework Help
Replies
2
Views
470
  • Special and General Relativity
Replies
8
Views
2K
  • Special and General Relativity
Replies
1
Views
670
  • Special and General Relativity
Replies
5
Views
1K
  • Special and General Relativity
2
Replies
59
Views
3K
  • Special and General Relativity
2
Replies
35
Views
2K
Replies
13
Views
640
  • Special and General Relativity
Replies
30
Views
5K
  • Special and General Relativity
Replies
17
Views
1K
Replies
3
Views
621
Back
Top